12
$\begingroup$

There have been many results on the first sign change of $\pi(x)-{\mathrm{li}}(x)$: among others, Lehman, te Riele, Bays & Hudson, Demichael, Chao & Plymen, and most recently Saouter & Demichel. These provide upper bounds (as well as lower bounds on some region in which $\pi(x)>{\mathrm{li}}(x)$).

Could these methods be used to generate a lower bound? That is, a region [2, y] in which $\pi(x)<{\mathrm{li}}(x)$. (Or even a region [x, y] where y is smaller than the best known upper bound, such that, in principle at least, direct calculations could yield a lower bound.)

It seems unlikely that direct searches like [2] will ever be able to resolve the exact value of the first crossing.


[1] C. Bays and R. H. Hudson. "A new bound for the smallest x with $\pi(x)>{\mathrm{li}}(x)$" Mathematics of Computation 69 (2000), pp. 1285–1296.

[2] T. Kotnik. "The prime-counting function and its analytic approximations", Advances in Computational Mathematics 29 (2008), pp. 55-70.

[2] R. Sherman Lehman. "On the difference $\pi(x)-{\mathrm{li}}(x)$" Acta Arithmetica 11 (1965), pp. 397–410.

[3] H. J. J. te Riele. "On the sign of the difference $\pi(x)-{\mathrm{li}}(x)$" Mathematics of Computation 48 (1987), pp. 323–328.

[4] Yannick Saouter and Patrick Demichel. "A sharp region where $\pi(x)-{\mathrm{li}}(x)$ is positive" Mathematics of Computation 79 (2010), pp. 2395-2405.

$\endgroup$
2
  • $\begingroup$ Just to be clear: is the lower limit of your definition of $\mathrm{li}$ 0 (i.e., Cauchy principal value integral) or 2? $\endgroup$ Dec 7, 2010 at 14:00
  • $\begingroup$ @J. M.: I was taking the Cauchy principal value where li(2) = 1.045..., but I would be interested in an answer to either, provided of course that x >= 8 to avoid small answers. $\endgroup$
    – Charles
    Dec 7, 2010 at 15:11

1 Answer 1

4
$\begingroup$

There are some explicit results in the work S. B. Stechkin, A. Yu. Popov, “The asymptotic distribution of prime numbers on the average”, Uspekhi Mat. Nauk, 51:6(312) (1996), 21–88

$\endgroup$
1
  • 1
    $\begingroup$ Thank you. I found the translation S. B. Stechkin and A. Yu Popov, Russ. Math. Surv. 51 (1996), pp. 1025-1092. $\endgroup$
    – Charles
    Nov 12, 2012 at 15:54

Your Answer

By clicking “Post Your Answer”, you agree to our terms of service and acknowledge you have read our privacy policy.

Not the answer you're looking for? Browse other questions tagged or ask your own question.